Бит поляков в пропагаторе

Привет, я пытаюсь вывести распространитель KG и застрял на том месте, где требуется интегральная формула Коши, т.е. оценка из

д 3 п ( 2 π ) 3 { 1 2 Е п е я п . ( Икс у ) | п о "=" Е п + 1 2 Е п е я п . ( Икс у ) | п о "=" Е п }
к
д 3 п ( 2 π ) 3 д п 0 я 2 π 1 п 2 м 2 е я п . ( Икс у )

Я так понимаю формула г ( г 0 ) "=" 1 2 π я г ( г ) г г 0 д г нужно использовать, но я просто не понимаю, как можно найти решение,

Я бы посоветовал сначала пойти наоборот. Если вы выполняете д п 0 интеграции вы получите трехмерный интеграл, который соответствует тому, с чего вы начали. Таким образом, вам в основном нужно «обратить» сложную интеграцию, что немного больно в первый раз, когда вы ее видите.
Этот вопрос очень неполный, но потенциально интересный, если его прояснить. Так можно немного уточнить?
@ChrisWhite спасибо за эту информацию, эта версия теперь выглядит хорошо и достаточно ясно, чтобы ее можно было оставить открытой.

Ответы (1)

По рецепту Фейнмана полюса расположены на п 0 "=" ± ( Е п я ϵ ) . Когда Икс 0 > у 0 , замкнем счетчик ниже положительного полюса так, что ( я п 0 ( Икс 0 у 0 ) ) < 0 ; Когда Икс 0 < у 0 , замкнем счетчик над отрицательным полюсом так, что ( я п 0 ( Икс 0 у 0 ) ) < 0 . По лемме Йодана мы знаем, что

| п 0 | "=" + д п 0 2 π я 1 п 2 м 2 е я п 0 ( Икс 0 у 0 ) "=" 0

Заметить, что ( п 0 ) 2 Е п 2 "=" ( п 0 Е п ) ( п 0 + Е п ) и счетчик, который мы выбираем, имеет только один полюс. Для Икс 0 > у 0 , у нас есть

г 0 "=" Е п , г ( г ) "=" 1 п 0 + Е п е я п 0 ( Икс 0 у 0 )

и для Икс 0 < у 0 , у нас есть

г 0 "=" Е п , г ( г ) "=" 1 п 0 Е п е я п 0 ( Икс 0 у 0 )

Тогда по теореме о вычетах

1 2 π я д п 0 г ( г ) г г 0 "=" г ( г 0 )

мы можем получить это

1 2 π я д п 0 1 ( п 0 ) 2 Е п 2 + я ϵ е я п 0 ( Икс 0 у 0 ) "=" 1 2 Е п е я Е п ( Икс 0 у 0 ) θ ( Икс 0 у 0 ) + 1 2 Е п е я Е п ( Икс 0 у 0 ) θ ( у 0 Икс 0 )

извините, я все еще немного запутался, я не понимаю, как вы сделали последний интеграл. Я понимаю, что p можно разделить на п 0 и его пространственные члены, но я все еще не понимаю, как сделать последний интеграл
Извините, я запутался с условностями, во-первых, это Е п 2 "=" м 2 + п я 2 , также не должны ли экспоненты иметь п не Е п , я действительно не понимаю, почему существует п 2 м 2 , извините за неловкость
п 2 "=" Е п 2 п я 2 "=" м 2 это состояние на оболочке. Обратите внимание, что е я п я ( Икс я у я ) появляется в трехмерном интеграле д 3 п